LSAT and Law School Admissions Forum

Get expert LSAT preparation and law school admissions advice from PowerScore Test Preparation.

User avatar
 Dave Killoran
PowerScore Staff
  • PowerScore Staff
  • Posts: 5850
  • Joined: Mar 25, 2011
|
#2155
Finally, in reference to June 2001, Section 2, Question #25 (the wood-burning stoves question), I think the problem here is with your assessment of the stimulus and answer choice (C). Let's look at each answer you mentioned briefly:

Answer choice (C): I don't see this as "cause with no effect." This would be better stated as "open fireplaces have effects that you failed to consider that are more severe than wood-burning fireplaces." Again, I believe that what you are doing is seeing elements of causality and then painting the whole problem with a simplified picture of what is occurring. In this case, the author focuses on one aspect of stoves/fireplaces: chimneys. On the basis of an analysis of chimney danger only, wood-burning stoves are said to be more dangerous. But is a chimney the only aspect of a stove/fireplace? No, and (C) points this out. When the author says in a wholesale way that "wood-burning stoves...are more dangerous," then the arguments can't be limited by "in the home" because that's part of the stove/fireplace environment.

In many respects, what's occurred here on your part is extremely similar to the tides/gravity question earlier in this thread. There, you missed the "induce" aspect and tried to make a causal argument about the whole relationship. To survive these questions you have to read the entire relationship and not focus in on just one part of it. When you do focus just on certain elements and ignore important elements, your chances of missing the problem rise dramatically.

Another issue is that you keep trying to argue with the test makers. While I can surely understand that desire :D , you have to stop doing that and instead start seeing the world from their perspective. When you miss a problem, instead of arguing for your answer as correct and theirs as wrong, you need to simply accept that they say you are wrong and start trying to understand why they think they are right. You somewhat came to see their point near the end of your post but you still wanted to argue out the "inside the home" limit instead of stopping and understanding why that wasn't relevant. If you focus on their viewpoint, you will find that many of the problems start to fall into recognizable categories. That said, on to the other two answers.

Answer choice (B): This answer doesn't tell us anything. Sure it depends on certain factors, but what does that means for open fireplaces or stoves? It could help or hurt, and that type of answer is never correct in a Weaken question.

Answer choice (A): This is a tricky answer because at first glance you might think it counters the argument. But the argument was based on "the smoke that wood-burning stoves release up the chimney is cooler...and travels more slowly and deposits more creosote (italics added for emphasis)." So, while according to (A) it may start out with less creosote than the open fireplace, because the smoke is cooler and travels more slowly, it can ultimately leave more of a deposit. The author can therefore easily counter this answer.

Please let me know if that helps. Thanks!
 jared.xu
  • Posts: 65
  • Joined: Oct 07, 2011
|
#2160
Thank you so much for your detailed explanations. I definitely learned something concrete from your third posting on June 2001 S2 Q25. I originally thought that the reason A was not right was because of the qualifier "the most efficient." And then I thought if A is wrong, then C should also be wrong if it uses the qualifier "inside the home." But your explanations clearly shows how the qualifier in A is not the reason it is wrong at all. You have now taught me that in a weaken question, the correct answer could contain qualifiers that limit the general scope of the stimulus and still manage to weaken it, and thus be correct. Even though the stimulus in this question compares wood-burning stoves and open fireplaces in general, the correct answer that weakens the argument could potentially contain words like "the most efficient wood-burning stoves" or "open fireplaces inside the home," right? Although the facts in answers A and C are about a limited number of wood-burning stoves or open fireplaces, they still weaken the argument that has a general scope because showing that a subset is not true would weaken the general statement about the whole. Is my analysis correct?

Your posting about the October 2003 Reading Comp Section, question #20, on the other hand, seems to show that I have problems with a conceptual understanding of what profitability means. The example you came up with clearly illustrates the position that you and the testmakers take. My understanding of the concept "profitability," however, is the state of yielding profit. My understanding of "profit" is the excess of returns over expenditure. So although your example is crystal clear to me, it does not seem to completely illustrate my idea of answer A: "The profitability of the India project was due primarily to temporary subsidies from the Indian government." If profitability was really due to temporary subsidies, I would have to revise your example thus:
Statement: "Tom's new lemonade stand had a profit of $50 last week!"
Response: "That seems about right. He made a total of $150 last week, and since Tom's Dad gave him a temporary subsidy of $100 last week to get by, we should subtract that expenditure from his overall return."
My issue is that in your previous example, we can't talk about "profitability" of Tom's lemonade stand due to his Dad's temporary subsidies at all because not only did he not have profits, he lost money. How could one talk of "profitability due to temporary subsidies" if there is not profit or return at all? For that reason, when I read the answer choice A, I was thinking of the India project making some profits (whether a little or a lot) with the help of temporary subsidies from the government. It never crossed my mind that the so-called "profitability" is merely the taking in of government money and treating it as "profit." And for that reason, in my mind, the author could retort this "weakening answer" and say: "because the Danish agency 'recognized' that the local government was going to invest along with them (local involvement), this project has a good chance of remaining competitive and profitable for the long run." Please tell me what you think, and how to correct my possible conceptual error. Thank you.
User avatar
 Dave Killoran
PowerScore Staff
  • PowerScore Staff
  • Posts: 5850
  • Joined: Mar 25, 2011
|
#2171
Hi Jared,

With the stoves question, I'm not in agreement with you that answer choice (C) has qualifications on its applicability. The stimulus says that wood burning stoves are more dangerous than open fireplaces. Assuming that the stimulus is referencing fireplaces/stoves inside the home (which appears justified with the talk of chimneys), answer choice (C) applies to all open fireplaces in that context, and shows that the danger claim isn't necessarily the case. The "most efficient" is definitely a qualifier, but that answer is wrong for a reason other than the qualifier. Otherwise, I agree with most of your analysis.

As far as whether a qualified statement will weaken an argument, it can, but it depends on context and the scope of the conclusion. For example, if you have a conclusion that says that "every lawyer in this firm is famous," all you need is a single example, qualified or otherwise, to refute that claim. On the other hand, if the conclusion is less broad, such as "some lawyers in this firm are famous," then you need a stronger/broader answer showing that none are famous. Thus, you can't create a single rule for Weaken questions; it depends on the nature of the statements in the stimulus. This is why the LSAT is difficult :D

For the RC question, you are kind of arguing for your interpretation again! Instead, I'd really focus on what they want you to understand in this problem. They make the point in the answer choice that "the profitability was due primarily to temporary subsidies." That's what they said, and they tell you to take that as true. Thus, I don't need/want to worry about accounting issues at that point because the test makers already defined the issue for me.

If you go back to the passage itself, the very last words are: "the project has a good chance of remaining competitive and profitable for the long run." If the subsidy is temporary, and was key to the initial profitability, don't you think that will undermine that long term-profitability chance?

Going back to the lemonade stand example, if Tom's father hadn't subsidized the lemonade stand, how would its long term-profitability look? At that point, not good. The point is that the temporary subsidy can alter the situation such that it artificially appears profitable when in fact without the subsidy it might not be. In our passage, when that temporary subsidy is pulled, things would probably look very different for the project.

Thanks!
 moshei24
  • Posts: 465
  • Joined: Mar 20, 2012
|
#5426
Hi,

I understand why answer A works, but for why doesn't answer E work? Isn't an assumption in the argument that correlation implies causation? Isn't the argument trying to prove that that isn't the case here? So if it's trying to provide an argument against that, wasn't that an initial assumption (faulty one)?
 moshei24
  • Posts: 465
  • Joined: Mar 20, 2012
|
#5427
Oh, I'm sorry - I think I just realized the answer to my question. They never actually assumed that correlation meant causation. They were trying to figure out if it did in their argument, and they concluded that it didn't. Is that correct?
 Joshua Kronick
PowerScore Staff
  • PowerScore Staff
  • Posts: 13
  • Joined: Jul 06, 2012
|
#5501
Correct, I think you got it!
 moshei24
  • Posts: 465
  • Joined: Mar 20, 2012
|
#5518
Thanks!
 lsatstudier
  • Posts: 49
  • Joined: Oct 24, 2016
|
#30373
What is the flaw here? Thank you!
 Adam Tyson
PowerScore Staff
  • PowerScore Staff
  • Posts: 5153
  • Joined: Apr 14, 2011
|
#30411
It's a type of causal flaw, Studier - the author is looking at correlation, then showing that the correlation is less than perfect, and concluding that there is no causation happening. That's kind of backwards from what we typically see - usually it's that there IS a correlation and the author improperly determines that there IS causation at work. However, just because the argument is backwards from our typical situation doesn't take this out of the causal bucket altogether - causality and correlation are still at work here.

The problem is that the author assumes no causation because of imperfect correlation. That means he failed to take into account two possible problems - 1) it may be that the damage, or some form of the damage, doesn't always cause the problem, and 2) it may be that some other thing may sometimes cause the problem even in the absence of the damage.

Hope that helps!
 Oakenshield
  • Posts: 19
  • Joined: Jul 08, 2016
|
#30554
Hi staff,
I have read the replies above. Good explanation!

I chose C (#11) because I thought words like "there are people" and "some people" were so weak that the sample appeared unrepresentative. In my opinion, that a minority of people fails to have the correlation doesn't make the causation unwarranted.

But the teacher said a classic "causation" in LSAT means "A always causes B". I think it means if causation indeed exists, some so-called "counterexamples" can't overturn such a causation at all.

If "unrepresentative sample" is the flaw, it implies such counterexamples indeed exist, in turn the causation becomes problematic. However, our task is to prove that causal connection can be justified, so A) is a good choice because it indicates it is still safe to say A always causes B.

Am I right? If I am wrong, could you tell me why C is not correct?

Thanks in advance.

Get the most out of your LSAT Prep Plus subscription.

Analyze and track your performance with our Testing and Analytics Package.